Integral help (substitution and boundaries)

  • Thread starter Thread starter Kqwert
  • Start date Start date
  • Tags Tags
    Calculus Integral
Click For Summary
SUMMARY

The discussion focuses on the evaluation of the improper integral \int_{0}^{1} \frac{\ln x}{x} \, dx and the appropriate bounds for substitution. Participants clarify that when using the substitution u = \ln x, the limits should be transformed accordingly, resulting in u(0) = \infty and u(1) = 0. It is emphasized that the proper definition of the integral involves taking the limit as h approaches 0 from the positive side, specifically \lim_{h \to 0+} \int_h^1 \frac{\ln x}{x} \, dx. The discussion also highlights the alternative approach of using the reverse chain rule to avoid complications with changing limits.

PREREQUISITES
  • Understanding of improper integrals
  • Knowledge of substitution methods in calculus
  • Familiarity with the natural logarithm function
  • Experience with limits in calculus
NEXT STEPS
  • Study the evaluation of improper integrals in detail
  • Learn about substitution techniques in integral calculus
  • Explore the reverse chain rule and its applications
  • Investigate the properties of the natural logarithm function
USEFUL FOR

Students and educators in calculus, particularly those focusing on integral calculus, as well as mathematicians interested in the nuances of improper integrals and substitution methods.

Kqwert
Messages
160
Reaction score
3

Homework Statement


Hello,
I´ve added an image where the task is shown. I am wondering about the integral bounds here. When doing the substitution, shouldn't the bounds be inf and 0? When using that u = ln x, i.e. u(0) =inf, u(1) = 0.

Homework Equations

The Attempt at a Solution

 

Attachments

  • Skjermbilde 2018-11-17 kl. 12.21.08.png
    Skjermbilde 2018-11-17 kl. 12.21.08.png
    10.9 KB · Views: 412
Physics news on Phys.org
Kqwert said:

Homework Statement


Hello,
I´ve added an image where the task is shown. I am wondering about the integral bounds here. When doing the substitution, shouldn't the bounds be inf and 0? When using that u = ln x, i.e. u(0) =inf, u(1) = 0.

If they are simply going step-by-step and changing the limits, then you are correct. Otherwise, there may be some intermediary steps to get to that point where they have further manipulated the integral.

Just wondering why use substitution when you can use reverse chain rule? Then you don't have to worry about the limits changing...

\int_{0}^{\infty} \frac{ln(x)}{x} = \frac{1}{2} ( ln(\infty)^2 - ln(0)^2)
 
Kqwert said:

Homework Statement


Hello,
I´ve added an image where the task is shown. I am wondering about the integral bounds here. When doing the substitution, shouldn't the bounds be inf and 0? When using that u = ln x, i.e. u(0) =inf, u(1) = 0.

Homework Equations

The Attempt at a Solution


The image is guilty of sloppy reasoning. The actual definition of the improper integral involved is
$$\int_0^1 \frac{\ln x}{x} \, dx \equiv \lim_{h \to 0+} \int_h^1 \frac{\ln x}{x} \, dx.$$
You can evaluate the integral ##\int_h^1 \ln(x)/x \, dx## and then examine the limit.
 
Question: A clock's minute hand has length 4 and its hour hand has length 3. What is the distance between the tips at the moment when it is increasing most rapidly?(Putnam Exam Question) Answer: Making assumption that both the hands moves at constant angular velocities, the answer is ## \sqrt{7} .## But don't you think this assumption is somewhat doubtful and wrong?

Similar threads

Replies
5
Views
2K
  • · Replies 3 ·
Replies
3
Views
2K
  • · Replies 7 ·
Replies
7
Views
3K
  • · Replies 2 ·
Replies
2
Views
2K
Replies
2
Views
2K
  • · Replies 7 ·
Replies
7
Views
2K
  • · Replies 5 ·
Replies
5
Views
2K
  • · Replies 2 ·
Replies
2
Views
1K
  • · Replies 21 ·
Replies
21
Views
2K
  • · Replies 4 ·
Replies
4
Views
4K